Vous êtes sur la page 1sur 15

Inequalities (Unit 1 Unit 3)

Solutions to Exercises
Inequalities (Unit 1)
1.

By the AM-GM inequality, we have

1 + ai
1 ai , i.e. 1 + ai 2 ai for all i. Hence
2

2n = (1 + a1 )(1 + a2 )" (1 + an )

2 a1

)( 2

) (

a2 " 2 an

= 2n a1a2 " an

Dividing both side by 2n , we have 1 a1a2 " an , so that a1a2 " an 1 .

2.

By the AM-GM inequality, we have

Similarly, we have

a12
1 a12
a12
a
a
+

,
i.e.
2a1 a2 .

2
2
a2
a2
2 a2

a12
a2
2a1 a2 , 2 2a2 a3 and so on. Hence
a2
a3

a2
a12 a2 2
+
+ " + n (2a1 a2 ) + (2a2 a3 ) + " + (2an a1 )
a2 a3
a1
= (2a1 + 2a2 + " + 2an ) (a2 + a3 + " + an + a1 )
= a1 + a2 + " + an

Alternative Solution
Without loss of generality, assume a1 a2 " an . Then we have

a12 a2 2 " an 2 and

1 1
1
" .
a1 a2
an

Using the fact that Random Sum Reverse Sum, we have


a 2 a2 a 2
a2
a12 a2 2
+
+ " + n 1 + 2 + " + n = a1 + a2 + " + an .
a2 a3
a1
a1 a2
an

3.

Let x = 1 a , y = 1 b and z = 1 c .
Then a + b + c = 2 implies a = 2 b c = 2 (1 y ) (1 z ) = y + z .
Similarly, we have b = z + x and c = x + y .
Hence the original inequality becomes

( x + y )( y + z )( z + x)
8 , or ( x + y )( y + z )( z + x) 8 xyz .
xyz
1

x+ y
xy , i.e. x + y 2 xy .
2

By the AM-GM inequality, we have

Similarly, y + z 2 yz and z + x 2 zx .
Consequently, ( x + y )( y + z )( z + x) ( 2 xy )( 2 yz ) ( 2 zx ) = 8 xyz , completing the proof.

4.

Without loss of generality, assume a b c. Then

1
1
1
.

b+c a+c a+b


Using the fact that Direct Sum Random Sum, we have
a
b
c
b
c
a
+
+

+
+
b+c a+c a+b b+c a+c a+b
Taking another random sum, we have
a
b
c
c
a
b
+
+

+
+
.
b+c a+c a+b b+c a+c a+b
Adding the above two inequalities, we have
b
c b+c a+c a+b
a
2
+
+
+
+
= 3,

b+c a+c a+b b+c a+c a+b


so that

a
b
c
3
+
+
.
b+c a+c a+b 2

Alternative Solution
By the Cauchy-Schwarz inequality, we have
b
c
a
2
+
+

[ a (b + c) + b(c + a ) + c(a + b) ] (a + b + c) .
b+c a+c a+b
Hence it suffices to prove that
(a + b + c)2
3
.
a (b + c) + b(c + a ) + c(a + b) 2

By the AM-GM inequality,


2(a 2 + b 2 + c 2 ) = 2a 2 + 2b 2 + 2c 2 + 4ab + 4bc + 4ca
= (a 2 + b 2 ) + (b 2 + c 2 ) + (c 2 + a 2 ) + 4ab + 4bc + 4ca
2ab + 2bc + 2ca + 4ab + 4bc + 4ca
= 3[ a (b + c) + b(c + a) + c(a + b) ]
so that the desired inequality follows.
2

5.

Let b1 < b2 < " < bn be a permutation of a1 , a2 , , an in ascending order.


Since a1 , a2 , , an are distinct positive integers, we have bi i for all i.
Using the fact that 1

1
1
" 2 and that Random Sum Reverse Sum, we have
2
2
n
a1 +

a
b
a2
b
+ " + n2 b1 + 22 + " + n2
2
n
n
2
2
n
2
1+ 2 +" + 2
n
2
1
1
= 1+ +"+
n
2

Inequalities (Unit 2)

1.

Setting z = x + y and taking cube root on both sides, the original inequality becomes
2

x2 + y 2 + ( x + y ) 3 3 2 x 3 y 3 ( x + y ) 3 .
2

Now, using the AM-GM inequality twice, we have


x2 + y2 x2 + y2
2
+
+ ( x + y)
2
2
2
2
x +y
2 xy
2

+
+ ( x + y)
2
2
3
2
= ( x + y)
2
4
2
3
= ( x + y)3 ( x + y)3
2

x2 + y 2 + ( x + y ) =
2

3
2 xy
2

4
3

( x + y)3

= 3 3 2x 3 y 3 ( x + y )3

Hence the original inequality is proved.

2.

1
1
Without loss of generality, assume x y z . Let z = k . Then 0 k .
3
3
2
x+ y 1 k
Using the facts that x + y = + k and xy
= + , we get
3
2 3 2
2

x+ y
xy + yz + zx 3 xyz z ( x + y ) +
(1 3z )
2
2

1
2
1 k
= k + k + + ( 3k )
3
3
3 2
2 3
= + k3
9 4
2 31
+
9 43
1

Alternative Solution
Note that

xy + yz + zx 3xyz = xy (1 z ) + yz (1 x) + xz (1 y )
= xy ( x + y ) + yz ( y + z ) + xz ( z + x)
= x 2 ( y + z ) + y 2 ( x + z ) + z 2 ( y + x)
= x 2 (1 x) + y 2 (1 y ) + z 2 (1 z )
Since x 2 (1 x)

x
x
x
= (1 2 x) 2 0 , we have x 2 (1 x) .
4
4
4

Similarly, y 2 (1 y )

y
z
and z 2 (1 z ) . Consequently,
4
4

xy + yz + zx 3 xyz = x 2 (1 x) + y 2 (1 y ) + z 2 (1 z )

3.

x y z 1
+ + = .
4 4 4 4

After some trial, we find that equality holds when the two triangles on the left hand side are
similar, i.e. when
x
y
z
= = .
x' y' z'
This is clearly the equality condition for the Cauchy-Schwarz inequality. Therefore, we
attempt to use the Cauchy-Schwarz inequality to solve the problem.
Now we must express the area of a triangle in terms of its side lengths. Clearly, we should use
the Herons formula, which states that the area of a triangle with side lengths a, b, c is
s ( s a )( s b)( s c) ,

where s =

a+b+c
. Hence the original inequality becomes
2
4

s ( s x)( s y )( s z ) + 4 s '( s ' x ')( s ' y ')( s ' z ')


4 ( s + s ')( s + s ' x x ')( s + s ' y y ')( s + s ' z z ')

with s =

x+ y+z
x '+ y '+ z '
and s ' =
. Now, using the Cauchy-Schwarz inequality twice, we
2
2

have
4

s ( s x)( s y )( s z ) + 4 s '( s ' x ')( s ' y ')( s ' z ')


s ( s x) + s '( s ' x ') ( s y )( s z ) + ( s ' y ')( s ' z ')

( s + s ')( s x + s ' x ') ( s y + s ' y ')( s z + s ' z ')

= 4 ( s + s ')( s + s ' x x ')( s + s ' y y ')( s + s ' z z ')


and so the original inequality is proved.

4.

Let a 2004 = 1 a1 a 2 " a 2003 . Then a1 + a2 + " + a2004 = 1 and


a1a2 " a2003 (1 a1 a2 " a2003 )
a1a2 " a2004
.
=
(a1 + a2 + " + a2003 )(1 a1 )(1 a2 )" (1 a2003 ) (1 a1 )(1 a2 )" (1 a2004 )
By the AM-GM inequality,
(1 a1 )(1 a2 )" (1 a2004 )
= (a2 + a3 + " + a2004 )(a1 + a3 + " + a2004 )" (a1 + a2 + " + a2003 )

2003 2003 a2 a3 " a2004

)( 2003

2003

) (

a1a3 " a2004 " 2003 2003 a1a2 " a2003

= 20032004 a1a2 " a2004

Hence we have

a1a2 " a2004


1
.

(1 a1 )(1 a2 )" (1 a2004 ) 20032004

Furthermore, equality holds when a1 = a 2 = " = a 2004 =


Therefore, the answer is

1
.
20032004

ak 2

k =1 a k + bk
n

5.

1
.
2004

By the Cauchy-Schwarz inequality,


2

ak
2
n

a
Hence k n k =1 =
k =1 a k + bk
(ak + bk )
k =1

n
n
(a k + bk ) a k .
k =1
k =1

ak
n
k =1 = a k

n
k =1 2
2 ak

k =1

Alternative Solution
For real numbers a and b, we have (a + b) 2 ( 2 ab ) = 4ab , so
2

ab
a+b
. Hence

a+b
4

ak 2

k =1 a k + bk
n

n a k 2 + a k bk a k bk
=
k =1
a k + bk

n
n
ab
= a k k k
k =1
k =1 a k + bk

n
n
a + bk
ak k

4
k =1
k =1
n
n
2a
= ak k
k =1 4
k =1
n
a
= k
k =1 2

6.

Let x = a + b c , y = b + c a and z = c + a b . Then the original inequality becomes


x+ y+ z

x+ y
+
2

y+z
z+x
+
.
2
2

By the AM-GM inequality, we have


2

x+ y
x + y + 2 xy x + y
xy x + y x + y x + y
=
+

+
=

=
2
4
4
2
4
4
2

and hence

x+ y

x+ y
. Similarly, we have
2
y+ z

y+z
and
2

z+ x

z+x
.
2

Adding these three inequalities, we have


x+ y+ z

x+ y
+
2

y+z
z+x
+
,
2
2

thereby proving the original inequality.


Finally, equality in the above application of AM-GM inequality occurs if x = y , i.e. x = y .
Similarly we must have y = z and z = x . If x = y , then a + b c = b + c a , hence 2a = 2c
and a = c . Similarly, we must have a = b and b = c . That is, equality holds if and only if
a =b = c.

7.

By the Cauchy-Schwarz inequality, we have

(x

+ y 2 + z 2 )(12 + 12 + 12 ) ( x + y + z ) 2

which gives x + y + z 3 ( x 2 + y 2 + z 2 ) . On the other hand, the AM-GM inequality asserts


that
2

x 2 + y 2 + z 2 3( xyz ) 3 .

xy + yz + zx 3( xyz ) 3 and

Consequently, we have

xyz x + y + z + x 2 + y 2 + z 2

(x

+ y + z ) ( xy + yz + zx)
2

) xyz (
=

xyz

(x

3 ( x2 + y 2 + z 2 ) + x2 + y 2 + z 2

(x

+ y + z ) ( xy + yz + zx)

)(

3 +1

x2 + y2 + z 2

+ y 2 + z 2 ) ( xy + yz + zx)

3 + 1 xyz

x 2 + y 2 + z 2 ( xy + yz + zx)

3 + 1 xyz
1

3( xyz ) 3 3( xyz ) 3

8.

3 +1
3 3

3+ 3
9

By the rearrangement inequality, we have a 3 + b3 a 2b + ab 2 = ab ( a + b ) .


Similarly, we have b3 + c 3 bc ( b + c ) and c 3 + a 3 ca ( c + a ) . Consequently,
1
1
1
+ 3 3
+ 3
3
a + b + abc b + c + abc c + a 3 + abc
1
1
1

+
+
ab ( a + b ) + abc bc ( b + c ) + abc ca ( c + a ) + abc
3

9.

1
1
1
+
+
ab ( a + b + c ) bc ( a + b + c ) ca ( a + b + c )

1
abc

Let a 1 = x 2 , b 1 = y 2 and c 1 = z 2 for some positive non-negative x, y, z. Then the


original inequality becomes
x+ y+ z

(z

+ 1) ( x 2 + 1)( y 2 + 1) + 1 .

By the Cauchy-Schwarz inequality, we have


x + y = x 1 + 1 y ( x 2 + 1)( y 2 + 1) .
Similarly, we have
x + y + z ( x + y ) 2 + 1 ( z 2 + 1) ( x 2 + 1)( y 2 + 1) + 1 ( z 2 + 1)
and proof is complete.

10. (a) When n = 2 and x1 = x 2 = 1 , C


On the other hand, when C =

1(1)(12 + 12 ) 1
= .
8
(1 + 1) 4

1
, the inequality holds for all real numbers x1 , , xn 0
8

since
4
2
1
1


xi = xi
8 1i n
8 1i n

1
2
= xi + 2 xi x j
8 1i n 1i < j n

1
2
2 2 xi x j xi
8
1i < j n 1i n

2
= xi x j xi
1i < j n 1i n
=

x x

( x1 + x2 + " + xn )

x x

( xi + x j )

1i < j n

1i < j n

Hence the required least constant C is

(AM-GM inequality)

1
.
8

(b) Consider the term with i = 1 and j = 2 in the last two expressions in (a).
We have x1 x 2 ( x1 + x 2 + " + x n ) x1 x 2 ( x1 + x 2 ) .
2

This equality holds if and only if x3 = x 4 = " = x n .


As the choice of i and j is arbitrary, if any (n 2) of the xi s are zero, then equality in the
last inequality holds, and vice versa
When (n 2) of the xi s are zero, the inequality is reduced to the case of n = 2.
Consider the application of AM-GM inequality in (a).

Equality holds if and only if x1 + x 2 = 2 x1 x 2 , or ( x1 x 2 ) 2 = 0 , i.e. x1 = x 2 .


2

Hence equality of the original inequality holds if and only if any (n 2) of the xi s are
zero and the remaining two xi s are equal (possibly to zero).

10

Inequalities (Unit 3)

1.

a + b + c a + b + c a b + c a + b c
By Herons formula, T =

2
2
2
2
Putting this into the original inequality, the original inequality can be simplified as follows:
a 2 + b 2 + c 2 3(a + b + c)( a + b + c)(a b + c)(a + b c)
a 4 + b 4 + c 4 + 2a 2b 2 + 2b 2 c 2 + 2c 2 a 2 3 (b + c) 2 a 2 a 2 (b c) 2
a 4 + b 4 + c 4 + 2a 2b 2 + 2b 2 c 2 + 2c 2 a 2 3 2bc + (b 2 + c 2 a 2 ) 2bc (b 2 + c 2 a 2 )
a 4 + b 4 + c 4 + 2a 2b 2 + 2b 2 c 2 + 2c 2 a 2 3 [ 2a 2b 2 + 2b 2 c 2 + 2c 2 a 2 a 4 b 4 c 4 ]
4a 4 + 4b 4 + 4c 4 4a 2b 2 + 4b 2 c 2 + 4c 2 a 2
By the AM-GM inequality, we have

4a 4 + 4b 4 + 4c 4 = (2a 4 + 2b 4 ) + (2b 4 + 2c 4 ) + (2c 4 + 2a 4 )


( 2 2a 4 2b 4 ) + ( 2 2b 4 2c 4 ) + ( 2 2c 4 2a 4 )
= 4a 2b 2 + 4b 2 c 2 + 4c 2 a 2
thereby proving the last inequality and hence the original inequality. It is clear in the
application of the AM-GM inequality that equality holds if and only if a = b = c .

Alternative Solution
Without loss of generality, assume that the angle opposite the side a is acute. Suppose that the
altitude from this vertex, whose length we denote by h, is of distances m and n from the
remaining 2 vertices, with b = h 2 + m 2 and c = h 2 + n 2 . WLOG, assume that m n. Then
a=m+n or m-n.
For a=m+n, the original inequality becomes
( m + n) 2 + ( h 2 + m 2 ) + ( h 2 + n 2 ) 4 3

( m + n) h
.
2

Rewriting this as a quadratic equality in h, we have


h 2 3(m + n)h + (m 2 + mn + n 2 ) 0 .
The discriminant of the quadratic function on the left is
2

= 3(m + n) 4(1)(m 2 + mn + n 2 ) = (m n) 2 0 .
Since the coefficient of h 2 is positive, this means h 2 3(m + n)h + (m 2 + mn + n 2 ) 0 for all
h, as desired. Equality holds when m = n , which means b = c . By symmetry, we need
a = b = c.

For a=m-n, the argument is the same as above, with n replaced by n.


11

2.

Rewrite the given inequality as c 2 = a 2 + b 2 2ab cos 60 .


Hence we see that a, b, c are the side lengths of a triangle where the angle opposite the side
with length c is equal to 60.
In a triangle, a side opposite a larger angle is longer. Since 60 = 180 3, one other angle of
the triangle must be at least 60 and the remaining angle must be at most 60. In other words, if
we assume (without loss of generality) that a b , then we must have a c and b c .
From this, we see that a c is positive while b c is negative, so that (a c)(b c) 0 .

3.

By the Cauchy-Schwarz inequality, we have


2
BC CA AB
+
+

( BC PD + CA PE + AB PF ) ( BC + CA + AB ) .
PD PE PF

Hence
BC CA AB
( BC + CA + AB) 2
+
+

.
PD PE PF BC PD + CA PE + AB PF

The right hand side of the above inequality is a constant, since the numerator is the square of
the perimeter while the denominator is twice the area.
Equality holds if and only if
BC CA AB
:
:
= ( BC PD) : (CA PE ) : ( AB PF ) ,
PD PE PF

or PD = PE = PF . In other words, the expression in the question is minimum when (and only
when) P is the incentre of ABC.

4.

Let x =

AI
BI
CI
1
8
, y=
and z =
. The inequality to be proved is then xyz
.
CC '
AA '
BB '
4
27

Note that
x+ y+z
AI BI CI
=
+
+
AP BQ CR
[ ABI ] + [CAI ] [ BAI ] + [ BCI ] [CAI ] + [CBI ]
=
+
+
[ ABC ]
[ ABC ]
[ ABC ]
=

2 ([ ABI ] + [ BCI ] + [CAI ])

C
I

[ ABC ]

2[ ABC ]
[ ABC ]
=2
=

C
A

12

Hence the AM-GM inequality asserts that


8
x+ y+ z 2
xyz
,
= =
3

3 27
3

thereby proving the right-hand inequality.


To prove the left-hand inequality, we first make some
additional observation as follows. Let D, E, F be the
mid-points of BC, CA, AB respectively. We claim
that I lies in DEF. Assuming the claim, we draw a
line through I parallel to BC cutting AB at S. Since
ASI ~ ABA, we have
x=

AI
AS AF 1
=
>
= .
AA ' AB AB 2

Similarly, we have y >

1
1
and z > .
2
2

Now we return to the proof of the claim, namely, that I lies in DEF. Indeed, the angle
bisector theorem yields
AI AB
BA ' AB
and
,
=
=
IA ' BA '
A ' C AC
BA '
AB
AI AB + AC BC
and hence
=
>
= 1 by the triangle inequality. Hence
=
BC AB + AC
IA '
BC
BC
I is below EF in the figure, and the same is true with respect to DF and DE, thereby
establishing the claim.

so that

Consequently, we may write


x=

1
1
1
+ , y = + and z = +
2
2
2

for some , , > 0 . Then we have


1
1
1

xyz = + + +
2
2
2

1 1
1
= + ( + + ) + ( + + ) +
8 4
2
1 1
> + ( + + )
8 4
1 11
= +
8 42
1
=
4

and hence proving the left-hand inequality.


13

5.

Let AB = BC = CD = a and DE = EF =
FA = b. As shown in the figure, construct
equilateral triangles ABX and DEY. Since
AXB = DYE = 60, AX = XB = BD = a
and DY = YE = EA = b, the two hexagons
ABCDEF and AXBDYE are congruent
and so CF = XY.

B
H

Since
AXB + AGB = 180 = DHE + DYE,

AXBG and DYEH are cyclic quadrilaterals.


Hence by the Ptolemys theorem,

AB XG = AX BG + XB AG ,

which is equivalent to aXG = aBG + aAG , or


XG = BG + AG .

Similarly, we have YH = DH + EH and hence


AG + GB + GH + DH + HE = XG + GH + YH XY = EF .

6.

As shown in the figure, extend BC and EF to


draw a rectangle PQRS enclosing the hexagon.
Since opposite sides of the hexagon are parallel,
opposite angles are equal (i.e. A = D, B =
E and C = F). Let a, b, c, d, e, f denote the
lengths of AB, BC, CD, DE, EF and FA
respectively. We have

D
S

2 BF PA + AS + QD + DR
= a sin B + f sin F + c sin C + d sin E
= a sin B + f sin C + c sin C + d sin B
Hence
RA =

BF
1 a sin B f sin C c sin C d sin B
=
+
+
+
.
2sin A 4 sin A
sin A
sin A
sin A

Similarly, we have
14

1 c sin A b sin B e sin B f sin A


RC =
+
+
+

4 sin C
sin C
sin C
sin C
and
1 e sin C d sin A a sin A b sin C
+
+
+
RE =
.
4 sin B
sin B
sin B
sin B
Adding these inequalities and using the fact that

x y
x y
+ 2
= 2 for x, y > 0 , we have
y x
y x

1
p
RA + RC + RE (2a + 2b + 2c + 2d + 2e + 2 f ) =
4
2
as desired.

15

Vous aimerez peut-être aussi